Last visit was: 23 Apr 2024, 11:12 It is currently 23 Apr 2024, 11:12

Close
GMAT Club Daily Prep
Thank you for using the timer - this advanced tool can estimate your performance and suggest more practice questions. We have subscribed you to Daily Prep Questions via email.

Customized
for You

we will pick new questions that match your level based on your Timer History

Track
Your Progress

every week, we’ll send you an estimated GMAT score based on your performance

Practice
Pays

we will pick new questions that match your level based on your Timer History
Not interested in getting valuable practice questions and articles delivered to your email? No problem, unsubscribe here.
Close
Request Expert Reply
Confirm Cancel
SORT BY:
Date
Tags:
Show Tags
Hide Tags
Math Expert
Joined: 02 Sep 2009
Posts: 92877
Own Kudos [?]: 618557 [10]
Given Kudos: 81561
Send PM
Current Student
Joined: 24 Oct 2016
Posts: 166
Own Kudos [?]: 227 [0]
Given Kudos: 116
Location: India
Concentration: Technology, Strategy
GMAT 1: 710 Q49 V38
GMAT 2: 760 Q50 V44 (Online)
GPA: 3.61
Send PM
Intern
Intern
Joined: 11 Nov 2020
Posts: 45
Own Kudos [?]: 8 [0]
Given Kudos: 82
Send PM
Intern
Intern
Joined: 02 Sep 2020
Posts: 9
Own Kudos [?]: 9 [0]
Given Kudos: 7
Location: India
GMAT 1: 700 Q49 V35
GMAT 2: 690 Q50 V33
GMAT 3: 730 Q49 V39
GPA: 3.3
Send PM
Re: Some people have been promoting a new herbal mixture as a remedy for [#permalink]
Please explain how C was eliminated.
Quote:
(C) showing that conditions necessary to establish the truth of a claim are met

Here the condition is: Nobody having cold buys the medicine.
This condition is used to establish the truth of the claim: Not effective.
Am I missing something?
Intern
Intern
Joined: 11 Nov 2020
Posts: 45
Own Kudos [?]: 8 [0]
Given Kudos: 82
Send PM
Re: Some people have been promoting a new herbal mixture as a remedy for [#permalink]
RaevMayfly wrote:
Please explain how C was eliminated.
Quote:
(C) showing that conditions necessary to establish the truth of a claim are met

Here the condition is: Nobody having cold buys the medicine.
This condition is used to establish the truth of the claim: Not effective.
Am I missing something?


I agree. That’s why I had chosen C. LSAT says A is the answer. The only way you can eliminate C is by assuming that there is just one claim and that is of the proponents of the herbal medicines

Posted from my mobile device
User avatar
Non-Human User
Joined: 01 Oct 2013
Posts: 17204
Own Kudos [?]: 848 [0]
Given Kudos: 0
Send PM
Re: Some people have been promoting a new herbal mixture as a remedy for [#permalink]
Hello from the GMAT Club VerbalBot!

Thanks to another GMAT Club member, I have just discovered this valuable topic, yet it had no discussion for over a year. I am now bumping it up - doing my job. I think you may find it valuable (esp those replies with Kudos).

Want to see all other topics I dig out? Follow me (click follow button on profile). You will receive a summary of all topics I bump in your profile area as well as via email.
GMAT Club Bot
Re: Some people have been promoting a new herbal mixture as a remedy for [#permalink]
Moderators:
GMAT Club Verbal Expert
6917 posts
GMAT Club Verbal Expert
238 posts
CR Forum Moderator
832 posts

Powered by phpBB © phpBB Group | Emoji artwork provided by EmojiOne